Answer:
Step-by-step explanation:
Measia rode her bike 174 miles in six days, riding the same distance each day. Jamal rode his bike 135 miles in five
days, riding the same distance each day. Which of the following statements is true?
Answer:
6
Step-by-step explanation:
, , , , ,, , , , , , , , ,sssss
Esmeralda has a magnet in the shape of a square. Each side of the magnet is 3 inches long. What is the perimeter of her magnet?
Answer:
12 inches
Step-by-step explanation:
This is a square of sidee length 3 inches. As there are four (equal) sides, the perimeter of this square magnet is 4(3 inches), or 12 inches.
4. A plumber charges $65 per hour to fix a leak. What is an equation that relates the total cost of a leak repair c to the number of hours, h, it takes the
plumber to fix the leak? Show work
Answer:
c = 65h.
Step-by-step explanation:
cost = rate per hour * hours worked:
c = 65 * h .
65, 46, 78, 3, 87, 12, 99, 38, 71, 38
what is the median??
Answer:
median = 55.5
Step-by-step explanation:
Here we have 10 numbers which is an even number
then the median for the data set is the average of the two middle numbers
firstly ,we have to put the number of the set in order from least to greatest like this :
3 < 12 < 38 < 38 < 46 < 65 < 71 < 78 < 87 < 99
———————— ————————-
As you can see the middle two numbers are : 46 and 65
Now, we calculate their average :
[tex]\large \text Median = \frac{46+65}{2}= 55.5[/tex]
Which phrase describes the linear relationship between the values of x and y shown in the table? A. y is 4 more than x B. y is 6 more than x C. y is 3 times x D. x is 3 times y
The linear relationship on the table has a constant slope
The linear relationship is (c) y is 3 times x
How to determine the linear relationship?The table of values is given as:
x l y
3 l 9
0 l 0
For both x and y values on the table, we have:
y = 3x
i.e.
3 * 3 = 9 and 3 * 0 = 0
This means that the linear relationship is (c) y is 3 times x
Read more about linear functions at:
https://brainly.com/question/14323743
find the measures of
Find the measure of
Find the measure of
Answer:
15.
If we look at angle BOD, that would form a right angle (90°), given that one part of that whole angle is 30°, the other would be;
30° + x = 90°
-30 -30
x = 60°, therefore ∠BOC = 60°.
If we take a look at angle EOC, we can see that that angle is a straight angle (180°), we can also see that a right angle (90°) is a part of that angle alongside the angle we just previously found (60°). So all of those angles plus the unknown angle (AOE) which we will consider 'x' summed up would result in 180 degrees.
Now we set up the equation;
90° + 60° + x = 180°
150 + x = 180
-150 -150
x = 30°, therefore ∠AOE = 30°.
16.
The sum of the interior angles in a triangle will always equal 180°. (We can also confirm this using the formula (n - 2) x 180.)
Given two of the angles, we must add them and the unknown angle(D) which we will consider 'x' to make it result in 180°.
Now we set up the equation;
55° + 18° + x = 180°
73 + x = 180
-73 -73
x = 107°, therefore ∠EDF = 107°.
17.
To find angle P, we must first find the supplement of 34° because 34° and the angle beside forms a straight angle (180°).
Set up an equation;
34 + x = 180
-34 -34
x = 146°, now that we've found the supplement, we add this supplement with the other given angle (23°) because all three angles (unknown angle which we will consider x + 23 + 146) will equal 180°(sum of interior angles of triangle).
146° + 23° + x = 180°
169 + x = 180
-169 -169
x = 11°, therefore ∠QPR = 11°.
18.
Seeing that the bigger triangle has a 90° angle (indicated with a square), and two other equal angles(indicated with the two lines on both legs of the big triangle), we solve for those two missing equal angles in the bigger triangle which we will then use to solve for the smaller triangle's angle.
2x + 90° = 180°
-90 -90
2x = 90
/2 /2
x = 45°, so now we know the two angles in the bigger triangle excluding the right angle.
One of those equal angles is vertical to the smaller triangle, and vertical angles are congruent.
Hence, the angle vertical to the bigger triangle in the smaller triangle will be 45°.
Now we solve for ∠CDE.
Add the two angles and the missing angle to equal 180°.
Set up the equation;
86° + 45° + x = 180°
131 + x = 180
-131 -131
x = 49°, so ∠CDE = 49°.
A wheel with the 31 in. diameter has 651 rotations. Reflect on how different wheel sizes would change your answer. What would happen to the number of rotations if the circumference of the wheels were increased by 20%? For the wheel with 31 in. diameter, multiply its circumference 97.4 in. by 1.2 =________.
The number of rotations is reduced to 542.501 when the diameter of wheel is increased by 20 %.
How to compare rotations between two wheels of different diameter
The quantity of rotations (n), no unit, done by a wheel is equal by total traveled distance (s), in inches, divided by its circumference (p), in inches:
[tex]n = \frac{s}{p}[/tex]
[tex]n = \frac{s}{\pi\cdot D}[/tex] (1)
Where D is the diameter of the wheel, in inches.
If we know that n = 651 and D = 31 in, then the travelled distance is:
s = (651) · π · (31 in) (2)
s ≈ 63400.481 in
The travelled distance of the 31-in diameter wheel with 651 rotations is approximately 63400.481 inches.
By (2) we understand that the travelled distance is directly proportional to the diameter. Hence, if the diameter of the wheel is increased by 20 %, then we must multiply the diameter by 1.2 and divide the travelled distance by this result:
s ≈ 1.2 · π · (31 in)
s ≈ 116.867 in
[tex]n = \frac{63400.481\,in}{116.867\,in}[/tex]
[tex]n = 542.501[/tex]
The number of rotations is reduced to 542.501 when the diameter of wheel is increased by 20 %. [tex]\blacksquare[/tex]
To learn more on circumferences, we kindly invite to check this verified question: https://brainly.com/question/4268218
The first three steps in writing f(x) = 40x 5x2 in vertex form are shown. write the function in standard form. f(x) = 5x2 40x factor a out of the first two terms. f(x) = 5(x2 8x) form a perfect square trinomial. (eight-halves) squared = 16 f(x) = 5(x2 8x 16) – 5(16) what is the function written in vertex form?
The function f(x) = 5x² + 40x is a parabola. Then the vertex of the parabola will be at (–4, –80).
What is the parabola?It is the locus of a point that moves so that it is always the same distance from a non-movable point and a given line. The non-movable point is called focus and the non-movable line is called the directrix.
The function f(x) = 5x² + 40x
Then add and subtract 80, then we have
f(x) = 5x² + 40x + 80 – 80
f(x) = 5(x² + 8x + 16) – 80
f(x) = 5(x + 4)² – 80
The vertex of the parabola will be at (–4, –80).
More about the parabola link is given below.
https://brainly.com/question/8495504
The lengths of two sides of a triangle are shown.
Side 1: 3x^2 − 2x − 1
Side 2: 9x + 2x^2 − 3
The perimeter of the triangle is 5x^3 + 4x^2 − x − 3.
Part A: What is the total length of the two sides, 1 and 2, of the triangle? Show your work.(4 points)
Part B: What is the length of the third side of the triangle? Show your work. (4 points)
Part C: Do the answers for Part A and Part B show that the polynomials are closed under addition and subtraction? Justify your answer. (2 points)
Answer:
See below ↓↓↓
Step-by-step explanation:
Part A
Total length = Side 1 + Side 2
3x² - 2x - 1 + 9x + 2x² - 33x² + 2x² + 9x - 2x - 1 - 35x² + 7x - 4Part B
Length of 3rd side = Perimeter - [Side 1 + Side 2]
L = 5x³ + 4x² - x - 3 - [5x² + 7x - 4]L = 5x³ + 4x² - x - 3 - 5x² - 7x + 4L = 5x³ - x² - 8x + 1Part C
Yes, as the resulting polynomial has a finite value we can conclude that polynomials are closed under addition and subtraction.
Part A: The total length of the two sides of the triangle is [tex]5x^2+7x-4[/tex]. This is obtained by adding the given two sides.
Part B: The length of the third side is [tex]5x^3-x^2-8x+1[/tex]. This is obtained by subtracting the sum of two sides from the perimeter of the triangle.
Part C: Yes, the Part A and Part B answers show that the polynomials are closed under addition and subtraction. This is because the expressions have like terms.
Polynomials:These are the expressions that are formed with constants, coefficients, and variables. based on the highest degree of the variable in the expressions, the polynomials are classified into many types.Calculation:Given that, a triangle has two sides of length [tex]3x^2-2x-1[/tex] and [tex]9x+2x^2-3[/tex]
The perimeter of the triangle is [tex]5x^3+4x^2-x-3[/tex]
Part A:
To find the total length of two sides, adding the two side
⇒ [tex](3x^2-2x-1)+9x+2x^2-3\\[/tex]
⇒ [tex]5x^2+7x-4[/tex]
(adding the like terms w.r.t their sign)
Part B:
To find the length of the third side, subtract the sum of two sides from the perimeter.
⇒ [tex](5x^3+4x^2-x-3)-(5x^2+7x-4)[/tex]
⇒ [tex]5x^3-x^2-8x+1[/tex]
Part C:
From Part A and Part B, it is proved that the polynomials undergo addition and subtraction. Hence, it is justified.
Therefore, Part A, Part B, and Part C were obtained.
Learn more about polynomials here:
https://brainly.com/question/1315292
#SPJ2
A machine is supposed to fill bags with an average of 19.2 ounces of candy. The manager of the candy factory wants to be sure that the machine does not consistently underfill or overfill the bags . So the manager plans to conduct a significance test at the alpha = 0.10 significance level of H_{0} / mu = 19.2; H a : mu ne19.2 where mu = the true mean amount of candy (in ounces) that the machine put in all bags filled that day. The manager takes a random sample of 75 bags of candy produced that day and weighs each bag. The sample mean weight for the bags of candy was 19.28 ounces and the sample standard deviation was 0.81 ounce.
Find the P-value.
Answer: c. Step-by-step explanation:
if the price of small box is $20 and the price of large box is $45. what is the greatest amount of money he received when he sells all the boxes he has made? For this amount of money how many boxes of each size did he make
Answer:
in order to know the money that he receive first the type and amount of box must be described unless the question may not have answer.
The kite below is formed by four right triangles. If AB= 3, DE= 12, and AB = BC = DB, what is the area of the kite?
Answer:
50.46???
Step-by-step explanation: im bad at this so if it help ima be proud nut uhhh i tryed ...
A trapezoid has an area of 151.2 square centimeters. One base is 8.8 cm long. The height measures 10.5 cm. What is the length of the other base?
Answer:
x=20
Step-by-step explanation:
(1/2)*10.5*(8.8+x)=151.2
8.8+x=151.2*2/10.5
8.8+x=28.8
x=28.8-8.8
x=20
Answer: 20 cm
Step-by-step explanation:
area of trapeziod = ( (base + base) / 2) x height
151.2 = ( (8.8 + base) / 2 ) x 10.5
14.4 = ( (8.8 + base) / 2)
14.4 x 2 = 8.8 + base
28.8 - 8.8 = base
base = 20 cm
Pls help me !!
a sector of a circle has a central angle measure of 90°, and an area of 7 square inches. what is the area of the entire circle?
area of the circle = __ square inches
Explanation:
90° is 90/360 = 1/4 of a full 360° rotation. Meaning that if the central angle of the pizza slice is 90°, then we have four equal slices.
One slice is 7 square inches, so all four slices combine to 7*4 = 28 square inches
A $70,000 mortgage is $629.81 per month. What was the percent and for how many years?
9%, 20 years
9%, 30 years
7%, 20 years
9%, 25 years
The interest percent and the maturity period of the $70,000 mortgage with a monthly payment of $629.81 are A. 9%, 20 years.
How can the monthly payments be calculated?The monthly payments can be computed using the present value annuity table or formula.
However, to compute the percent interest and the maturity period, we can use an online finance calculator as below by inputting the calculated monthly payments.
Thus, at an interest rate of 9% for 20 years and a monthly payment of $629.81, the present value of $70,000 will be equal.
Data and Calculations;N (# of periods) = 240 (20 years x 12)
I/Y (Interest per year) = 9%
PMT (Periodic Payment) = $629.81
FV (Future Value) = $0
Results:
PV = $70,000.20
Sum of all periodic payments = $151,154.40 ($629.81 x 240)
Total Interest $81,154.20
The interest percent and the maturity period of the $70,000 mortgage with a monthly payment of $629.81 are A. 9%, 20 years.
Learn more about periodic payments at https://brainly.com/question/24244579
Mr. Blue's first year salary is $30,000 and it doubles each year. What is the explicit
rule for the sequence in simplified form? what is the recursive rule?
The explicit rule for the sequence in simplified form is y = 30000(2)^n
How to write the exponential functionThe standard exponential equation is expressed as:
y = ab^n
Given the following parameters
a is the initial values = 30,000
If the amount doubles each year then b = 2
Substitute the parameters
y = 30000(2)^n
Hence the explicit rule for the sequence in simplified form is y = 30000(2)^n
Learn more on explicit rule here: https://brainly.com/question/2059043
Answer: Explicit Rule: a_n=30,000 • 2^n-1 Recursive Rule: a_n = 2a_n-1; a_1 =30,000
Step-by-step explanation: the Explicit Rule for a geometric sequence is a_n = r • a_n -1 and the Recursive Rule is a_n = a_1 • a_n-1
a_1 is the first term and r is the common ration. In this case, the first term is 30,000 as stated in the question and the common ration is 2 since it doubles each time. So substitute those numbers into the formulas and that’s what you’ll get! Hope this helps! God bless you all!!
find the equation of the line drawn in the coordinate system below
Answer:
y=-2/1+4
or y=-2+4
Step-by-step explanation:
to find the slope, you count from one point of intersection to the next and the way it works is y/x so the line from one point to the next goes down 2 (making it negative also) and over 1, so -2/1 is your slope, and for the b part.. its just where the line intersects on the y axis (the line that goes up and down) so it would be 4
A betta fish tank is in the shape of a cube. The volume of the fish tank is 216
cubic inches. What is the edge length, in inches, of the fish tank?
6)
Type the number in the box.
inches
To find this answer, we need to understand how to get the Volume.
We get the volume by the formula: L x W x H
L = Length
W = Width
H = Height
So, we can see that there are 3 units required to know for getting the volume, if that's true, then, to do the inverse operation of finding the volume. That can be done by using thie formula:
[tex]V=a^3[/tex]
a= edge
Solving for a
a=V⅓=216⅓=6
So hence, each of the Cube shaped Fish Tank is 6in³now, let's look at how to find whether our answer is right or wrong
6x6=36
36x6=216
So hence, our answer is right!
-------------------------------------------------------------------------------------------------------"The beautiful thing about learning is that no one can take it away from you!" B.B.King
Thanks!
Mark me brainliest!
Have a GREAT AWESOME day!
~ms115~
:D
The length of a betta fish tank that has a shape of a cube and volume of 216 cubic inches is 6 inches.
Volume of a cube
A cube has all its sides equal to each other. Therefore,
volume of cube = L³
where
L = lengthTherefore,
L = >
volume = 216 cubic inches
volume of cube = L³
216 = L³
cube root both sides
∛216 = ∛L³
L = ∛216
L = 6 inches
learn more on volume here: https://brainly.com/question/10961243
What is the area of this triangle?
1375 ft
1500ft
3300ft
4500ft
Answer:
probably 20 man think about it
Step-by-step explanation:
find the volume
6ft
4ft
3ft
(i know it’s not 72)
Answer:
[tex] \fbox{Volume \: of \: pyramid \: = 24 ft³}[/tex]
Step-by-step explanation:
Given:
shape of object = Pyramidal
length of base l= 4ft
width of base w = 3ft
height of pyramid h = 6 ft
To find:
volume of pyramid = ?
Solution:
[tex]Volume \: of \: pyramid= \frac{l \times w \times h}{3} \\ Volume \: of \: pyramid= \frac{6 \times 4 \times 3}{3} \\ Volume \: of \: pyramid= \frac{24 \times \not3}{ \not3} \\ Volume \: of \: pyramid= 24 \: {ft}^{3} [/tex]
[tex] \fbox{Volume \: of \: pyramid \: = 24 ft³}[/tex]
Thanks for joining brainly community!True or False
1. A solution to 8 = -x + 10 is 2
Answer:
TrueStep-by-step explanation:
So, Lets find out if the solution is true or not, So first let us find the value of x.
⇒ 8 = -x + 10
We can rewrite the equation as,
⇒ -x + 10 = 8
Subtracting 10 from both sides we get :
⇒ -x + 10 – 10 = 8 – 10
⇒ -x = -2
Multiplying both sides by (-1) :
⇒ -1(-x) = -1(-2)
⇒ x = 2
Therefore,
The statement "A solution to 8 = -x + 10 is 2" is trueHey ! there
Answer:
TrueStep-by-step explanation:
In this question we are given with an equation that is 8 = -x + 10 and we have also given its solution that is 2 .And We are asked to tell whether the solution of given equation is true or false .
For finding that we must have to solve the equation .
Solution : -
[tex]\longrightarrow \qquad \: 8 = -x + 10[/tex]
Step 1 : Flipping the equation so that there's ease in solving :
[tex] \longrightarrow \qquad \: - x + 10 = 8[/tex]
Step 2 : Subtracting with 10 on both sides :
[tex] \longrightarrow \qquad \: - x + \cancel{10 }- \cancel{ 10} =8 - 10[/tex]
On further calculations , We get :
[tex] \longrightarrow \qquad \: - x = - 2[/tex]
Step 3 : Multiplying with -1 on both sides :
[tex] \longrightarrow \qquad \: - x \times - 1 = - 2 \times - 1[/tex]
We get :
[tex] \longrightarrow \qquad \: \blue{\underline{\boxed{ \frak{x = 2}}}}[/tex]
Therefore , it's TRUE that 2 is the solution of given equation .Alternative Solution : -
There's another easy way to check whether 2 is solution of given equation or not . This is done by substituting value of x that is 2 in given equation that is 8 = -x + 10 . So substituting value of x ( solution ) in given equation :
8 = - ( 2 ) + 108 = -2 + 108 = 8L.H.S = R.H.SAs we can see that left side is equal to right side .
Therefore , it's TRUE that 2 is the solution of given equation .#Keep Learning1/5c= 2 (whole number) 1/2
what is it
you have to divide them
ill also give the smartest
PLEASE
The value of c in the equation, 1/5c = 2 1/2 when divided is 12 1/2.
What are fractions?Fractions are numbers that are non integers. They are ususally made up of numerators and denominators. The numerator is the number above and the denominator is the number below.
What is the value of c?In order to determine the value of c, divide 2 1/2 by 1/5
2 1/2 ÷ 1/5
Convert the mixed fraction to an improper fraction
5/2 x 5 = 25/2 = 12 1/2
To learn more about the division of fractions, please check: https://brainly.com/question/25779356
Can someone please help me on this
Answer:
1. Colored men
2. Not really sure, but maybe on a wall ??
3. To notify colored men what the president proclaimed
Step-by-step explanation:
Which two sets of events are most likely independent?
O A. Event A: A student makes the drama team.
Event B. The student likes macaroni and cheese.
B. Event A: A student gets enough sleep.
Event B. The student is alert in class.
C. Event A: A car is blue.
Event B. The car lasts over 20 years.
D. Event A: A woman buys cat toys.
Event B: The woman has a cat.
Answer:
A - Event A: A student makes the drama team. / Event B. The student likes macaroni and cheese.C - Event A: A car is blue. / Event B. The car lasts over 20 years.What method would you choose to solve the equation 2x 2squared – 7 = 9? Explain why you chose this method.
Answer:
Step-by-step explanation:
(2x2squared-7)2squared=9 2squared
4x4squared-28x2squared+49=81
4x4squared-28x2squared=81-49
4x4squared-28x2squared=32
The blades of a windmill turn on an axis that is 35 feet above the ground. the blades are 10 feet long and complete two rotations every minute. which of the following equations can be used to model h, the height in feet of the end of one blade, as a function of time, t, in seconds? assume that the blade is pointing to the right, parallel to the ground at t = 0 seconds, and that the windmill turns counterclockwise at a constant rate. h = negative 10 sine (startfraction pi over 15 endfraction t) 35 h = negative 10 sine (pi t) 35 h = 10 sine (startfraction pi over 15 endfraction t) 35 h = 10 sine (pi t) 35
Answer:
it is c
Step-by-step explanation:
:)
The sine equation is [tex]\rm y = 10sin(\frac{\pi}{15} t)+35[/tex] an option (c) is correct.
It is given that the blades of a windmill turn on an axis that is 35 feet above the ground. the blades are 10 feet long and complete two rotations every minute.
It is required to model the sine equation.
What is trigonometry?Trigonometry is a branch of mathematics that deals with the relationship between sides and angles of a right-angle triangle.
We know the standard equation for the sine equation:
[tex]\rm y = Asin(wx)+h[/tex]
Where [tex]\rm T=\frac{2\pi}{w}[/tex]
[tex]\rm w = \frac{2\pi}{30}[/tex] ( In one minute, they complete two rotations.)
[tex]\rm w = \frac{\pi}{15}[/tex] ( T = 30)
At time t=0, the blade is horizontal. Hence, at t = 0, h = 35
And A = 10
Put all the values in the sine equation:
[tex]\rm y = 10sin(\frac{\pi}{15} t)+35[/tex]
Thus, the sine equation is [tex]\rm y = 10sin(\frac{\pi}{15} t)+35[/tex] an option (c) is correct.
Know more about trigonometry here:
brainly.com/question/26719838
The linear function f(x)=ax+b is graphed in the coordinate plane. Two equal intervals are compared,[-3,3] and [9,15]. What conclusion can be drawn from each interval of 6?
The linear function f(x) = ax + b has a constant rate
The conclusion from the each interval is that the function values changes at a constant rate
How to determine the conclusion?
The linear function is given as:
f(x) = ax + b
The interval is given as:
[-3,3] and [9,15]
Since the function is a linear function, the conclusion from the each interval is that the function values changes at a constant rate
Read more about linear functions at:
https://brainly.com/question/14323743
What is the y-intercept of the line shown? A. 2 B. -2 C. 4 D. -4
The y-intercept will be where it crosses the y-axis. The y-axis is the line that goes up and down also know a vertically.
it’s urgent please help
Answer:
x = 5.4
Step-by-step explanation:
The marked sides are the hypotenuse and the side adjacent to the marked angle. That suggests the appropriate trig function is the cosine:
Cos = Adjacent/Hypotenuse
cos(19°) = 5.1/x
x = 5.1/cos(19°) . . . . . multiply by x/cos(19°)
x ≈ 5.4
I don’t understand this at all
keeping in mind that Mt Everest is 8000 meters, or namely 8 Kilometers, Check the picture below.